$\begingroup$

Can someone please help me to find the answers and the logic behind the 2 questions below?

The questions are from the test taken from the website:
http://www.iq-brain.com/example/1

I could solve most of the questions, but the below ones I am not sure.

1:
enter image description here

2:
enter image description here

$\endgroup$

2